Difference between revisions of "2019 AMC 10B Problems/Problem 2"

Line 1: Line 1:
I couldn't if I tried
+
{{duplicate|[[2019 AMC 10B Problems|2019 AMC 10B #2]] and [[2019 AMC 12B Problems|2019 AMC 12B #2]]}}
 +
 
 +
==Problem==
 +
 
 +
Consider the statement, "If <math>n</math> is not prime, then <math>n-2</math> is prime." Which of the following values of <math>n</math> is a counterexample to this statement.
 +
 
 +
<math>\textbf{(A) } 11 \qquad \textbf{(B) } 15 \qquad \textbf{(C) } 19 \qquad \textbf{(D) } 21 \qquad \textbf{(E) } 27</math>
 +
 
 +
==Solution==
 +
 
 +
answer should be 27 (SuperWill)
 +
 
 +
==See Also==
 +
{{AMC10 box|year=2019|ab=B|num-b=1|num-a=3}}
 +
{{AMC12 box|year=2019|ab=B|num-b=1|num-a=3}}
 +
{{MAA Notice}}

Revision as of 13:20, 14 February 2019

The following problem is from both the 2019 AMC 10B #2 and 2019 AMC 12B #2, so both problems redirect to this page.

Problem

Consider the statement, "If $n$ is not prime, then $n-2$ is prime." Which of the following values of $n$ is a counterexample to this statement.

$\textbf{(A) } 11 \qquad \textbf{(B) } 15 \qquad \textbf{(C) } 19 \qquad \textbf{(D) } 21 \qquad \textbf{(E) } 27$

Solution

answer should be 27 (SuperWill)

See Also

2019 AMC 10B (ProblemsAnswer KeyResources)
Preceded by
Problem 1
Followed by
Problem 3
1 2 3 4 5 6 7 8 9 10 11 12 13 14 15 16 17 18 19 20 21 22 23 24 25
All AMC 10 Problems and Solutions
2019 AMC 12B (ProblemsAnswer KeyResources)
Preceded by
Problem 1
Followed by
Problem 3
1 2 3 4 5 6 7 8 9 10 11 12 13 14 15 16 17 18 19 20 21 22 23 24 25
All AMC 12 Problems and Solutions

The problems on this page are copyrighted by the Mathematical Association of America's American Mathematics Competitions. AMC logo.png